Question and Answers Forum

All Questions      Topic List

Others Questions

Previous in All Question      Next in All Question      

Previous in Others      Next in Others      

Question Number 18261 by Tinkutara last updated on 17/Jul/17

What is the maximum angle to the  horizontal at which a stone can be  thrown and always be moving away  from the thrower?

$$\mathrm{What}\:\mathrm{is}\:\mathrm{the}\:\mathrm{maximum}\:\mathrm{angle}\:\mathrm{to}\:\mathrm{the} \\ $$$$\mathrm{horizontal}\:\mathrm{at}\:\mathrm{which}\:\mathrm{a}\:\mathrm{stone}\:\mathrm{can}\:\mathrm{be} \\ $$$$\mathrm{thrown}\:\mathrm{and}\:\mathrm{always}\:\mathrm{be}\:\mathrm{moving}\:\mathrm{away} \\ $$$$\mathrm{from}\:\mathrm{the}\:\mathrm{thrower}? \\ $$

Answered by ajfour last updated on 26/Jul/17

r=(√(x^2 +y^2 ))   ,  x=u_x t ,   y=u_y t−((gt^2 )/2)  (dr/dt)=(((((2xdx)/dt)+((2ydy)/dt)))/(2(√(x^2 +y^2 ))))      =((u_x ^2 t+(u_y t−((gt^2 )/2))(u_y −gt))/(1(√(x^2 +y^2 ))))   tan θ=(u_y /u_x )  (dr/dt)>0 ⇒t[u_x ^2 +(u_y −((gt)/2))(u_y −gt)]>0  or     ((g^2 t^2 )/2)−((3u_y gt)/2)+u_x ^2 +u_y ^2 >0  ⇒ D=((9u_y ^2 g^2 )/4)−2g^2 (u_x ^2 +u_y ^2 ) < 0  ⇒       (u_y /(√(u_x ^2 +u_y ^2 ))) < ((2(√2))/3)  or    (u_y /u) < ((2(√2))/3)  ⇒   sin θ < ((2(√2))/3)  tan θ < 2(√2)      cos θ > (1/3)    θ < cos^(−1) (1/3) which is same as    θ < tan^(−1) (2(√2)) .

$$\mathrm{r}=\sqrt{\mathrm{x}^{\mathrm{2}} +\mathrm{y}^{\mathrm{2}} }\:\:\:,\:\:\mathrm{x}=\mathrm{u}_{\mathrm{x}} \mathrm{t}\:,\:\:\:\mathrm{y}=\mathrm{u}_{\mathrm{y}} \mathrm{t}−\frac{\mathrm{gt}^{\mathrm{2}} }{\mathrm{2}} \\ $$$$\frac{\mathrm{dr}}{\mathrm{dt}}=\frac{\left(\frac{\mathrm{2xdx}}{\mathrm{dt}}+\frac{\mathrm{2ydy}}{\mathrm{dt}}\right)}{\mathrm{2}\sqrt{\mathrm{x}^{\mathrm{2}} +\mathrm{y}^{\mathrm{2}} }} \\ $$$$\:\:\:\:=\frac{\mathrm{u}_{\mathrm{x}} ^{\mathrm{2}} \mathrm{t}+\left(\mathrm{u}_{\mathrm{y}} \mathrm{t}−\frac{\mathrm{gt}^{\mathrm{2}} }{\mathrm{2}}\right)\left(\mathrm{u}_{\mathrm{y}} −\mathrm{gt}\right)}{\mathrm{1}\sqrt{\mathrm{x}^{\mathrm{2}} +\mathrm{y}^{\mathrm{2}} }} \\ $$$$\:\mathrm{tan}\:\theta=\frac{\mathrm{u}_{\mathrm{y}} }{\mathrm{u}_{\mathrm{x}} } \\ $$$$\frac{\mathrm{dr}}{\mathrm{dt}}>\mathrm{0}\:\Rightarrow\mathrm{t}\left[\mathrm{u}_{\mathrm{x}} ^{\mathrm{2}} +\left(\mathrm{u}_{\mathrm{y}} −\frac{\mathrm{gt}}{\mathrm{2}}\right)\left(\mathrm{u}_{\mathrm{y}} −\mathrm{gt}\right)\right]>\mathrm{0} \\ $$$$\mathrm{or}\:\:\:\:\:\frac{\mathrm{g}^{\mathrm{2}} \mathrm{t}^{\mathrm{2}} }{\mathrm{2}}−\frac{\mathrm{3u}_{\mathrm{y}} \mathrm{gt}}{\mathrm{2}}+\mathrm{u}_{\mathrm{x}} ^{\mathrm{2}} +\mathrm{u}_{\mathrm{y}} ^{\mathrm{2}} >\mathrm{0} \\ $$$$\Rightarrow\:\mathrm{D}=\frac{\mathrm{9u}_{\mathrm{y}} ^{\mathrm{2}} \mathrm{g}^{\mathrm{2}} }{\mathrm{4}}−\mathrm{2g}^{\mathrm{2}} \left(\mathrm{u}_{\mathrm{x}} ^{\mathrm{2}} +\mathrm{u}_{\mathrm{y}} ^{\mathrm{2}} \right)\:<\:\mathrm{0} \\ $$$$\Rightarrow\:\:\:\:\:\:\:\frac{\mathrm{u}_{\mathrm{y}} }{\sqrt{\mathrm{u}_{\mathrm{x}} ^{\mathrm{2}} +\mathrm{u}_{\mathrm{y}} ^{\mathrm{2}} }}\:<\:\frac{\mathrm{2}\sqrt{\mathrm{2}}}{\mathrm{3}} \\ $$$$\mathrm{or}\:\:\:\:\frac{\mathrm{u}_{\mathrm{y}} }{\mathrm{u}}\:<\:\frac{\mathrm{2}\sqrt{\mathrm{2}}}{\mathrm{3}}\:\:\Rightarrow\:\:\:\mathrm{sin}\:\theta\:<\:\frac{\mathrm{2}\sqrt{\mathrm{2}}}{\mathrm{3}} \\ $$$$\mathrm{tan}\:\theta\:<\:\mathrm{2}\sqrt{\mathrm{2}}\:\:\:\:\:\:\mathrm{cos}\:\theta\:>\:\frac{\mathrm{1}}{\mathrm{3}} \\ $$$$\:\:\theta\:<\:\mathrm{cos}^{−\mathrm{1}} \left(\mathrm{1}/\mathrm{3}\right)\:\mathrm{which}\:\mathrm{is}\:\mathrm{same}\:\mathrm{as} \\ $$$$\:\:\theta\:<\:\mathrm{tan}^{−\mathrm{1}} \left(\mathrm{2}\sqrt{\mathrm{2}}\right)\:. \\ $$

Commented by Tinkutara last updated on 26/Jul/17

Thanks Sir!

$$\mathrm{Thanks}\:\mathrm{Sir}! \\ $$

Terms of Service

Privacy Policy

Contact: info@tinkutara.com